Let $f:(0;infty)times (0;infty) rightarrow Bbb R$ be a Lebesgue measurable function, is...












3












$begingroup$


Let $f:(0;infty)times (0;infty) rightarrow Bbb R$ be a Lebesgue measurable function. Lets define
$$T_f(y)=int_0^{+infty} f(x,y) , dy$$
$$G_f(y)=int_0^{+infty} |f(x,y)| , dy$$
By Tonelli's Theorem I know $G_f$ is measurable and I also know $G_f in L^p(0;+infty)$ (take it as an hypothesis). I would like to say $Vert T_f Vert _p le Vert G_f Vert _p$ so $T_f in L^p(0;+infty)$ but I need $T_f$ to be measurable in order to be abel to calculate $Vert T_f Vert _p$. Is this true? How can I prove it?










share|cite|improve this question









$endgroup$












  • $begingroup$
    $T_f$ is a pointwise limit of measurable functions (truncate the integral at $n$, then you can use the full Fubini theorem)
    $endgroup$
    – Mindlack
    Dec 15 '18 at 23:03










  • $begingroup$
    wich is the difference between "full Fubini Theorem" and "Fubini Theorem"?
    $endgroup$
    – Marcos Martínez Wagner
    Dec 15 '18 at 23:11










  • $begingroup$
    To me Fubini-Tonelli refers to the Fubini theorem when everything is non-negative (which is easier). By “full Fubini” I thought of the theorem when your function is not non-negative. So nothing really important.
    $endgroup$
    – Mindlack
    Dec 15 '18 at 23:13










  • $begingroup$
    Thank you!! I think I got it
    $endgroup$
    – Marcos Martínez Wagner
    Dec 15 '18 at 23:19










  • $begingroup$
    Nol, in fact the integral defining $T_f$ need not exist. (Yes if $f$ is integrable on the product space.)
    $endgroup$
    – David C. Ullrich
    Dec 15 '18 at 23:22
















3












$begingroup$


Let $f:(0;infty)times (0;infty) rightarrow Bbb R$ be a Lebesgue measurable function. Lets define
$$T_f(y)=int_0^{+infty} f(x,y) , dy$$
$$G_f(y)=int_0^{+infty} |f(x,y)| , dy$$
By Tonelli's Theorem I know $G_f$ is measurable and I also know $G_f in L^p(0;+infty)$ (take it as an hypothesis). I would like to say $Vert T_f Vert _p le Vert G_f Vert _p$ so $T_f in L^p(0;+infty)$ but I need $T_f$ to be measurable in order to be abel to calculate $Vert T_f Vert _p$. Is this true? How can I prove it?










share|cite|improve this question









$endgroup$












  • $begingroup$
    $T_f$ is a pointwise limit of measurable functions (truncate the integral at $n$, then you can use the full Fubini theorem)
    $endgroup$
    – Mindlack
    Dec 15 '18 at 23:03










  • $begingroup$
    wich is the difference between "full Fubini Theorem" and "Fubini Theorem"?
    $endgroup$
    – Marcos Martínez Wagner
    Dec 15 '18 at 23:11










  • $begingroup$
    To me Fubini-Tonelli refers to the Fubini theorem when everything is non-negative (which is easier). By “full Fubini” I thought of the theorem when your function is not non-negative. So nothing really important.
    $endgroup$
    – Mindlack
    Dec 15 '18 at 23:13










  • $begingroup$
    Thank you!! I think I got it
    $endgroup$
    – Marcos Martínez Wagner
    Dec 15 '18 at 23:19










  • $begingroup$
    Nol, in fact the integral defining $T_f$ need not exist. (Yes if $f$ is integrable on the product space.)
    $endgroup$
    – David C. Ullrich
    Dec 15 '18 at 23:22














3












3








3





$begingroup$


Let $f:(0;infty)times (0;infty) rightarrow Bbb R$ be a Lebesgue measurable function. Lets define
$$T_f(y)=int_0^{+infty} f(x,y) , dy$$
$$G_f(y)=int_0^{+infty} |f(x,y)| , dy$$
By Tonelli's Theorem I know $G_f$ is measurable and I also know $G_f in L^p(0;+infty)$ (take it as an hypothesis). I would like to say $Vert T_f Vert _p le Vert G_f Vert _p$ so $T_f in L^p(0;+infty)$ but I need $T_f$ to be measurable in order to be abel to calculate $Vert T_f Vert _p$. Is this true? How can I prove it?










share|cite|improve this question









$endgroup$




Let $f:(0;infty)times (0;infty) rightarrow Bbb R$ be a Lebesgue measurable function. Lets define
$$T_f(y)=int_0^{+infty} f(x,y) , dy$$
$$G_f(y)=int_0^{+infty} |f(x,y)| , dy$$
By Tonelli's Theorem I know $G_f$ is measurable and I also know $G_f in L^p(0;+infty)$ (take it as an hypothesis). I would like to say $Vert T_f Vert _p le Vert G_f Vert _p$ so $T_f in L^p(0;+infty)$ but I need $T_f$ to be measurable in order to be abel to calculate $Vert T_f Vert _p$. Is this true? How can I prove it?







real-analysis analysis lebesgue-integral lebesgue-measure






share|cite|improve this question













share|cite|improve this question











share|cite|improve this question




share|cite|improve this question










asked Dec 15 '18 at 23:00









Marcos Martínez WagnerMarcos Martínez Wagner

1148




1148












  • $begingroup$
    $T_f$ is a pointwise limit of measurable functions (truncate the integral at $n$, then you can use the full Fubini theorem)
    $endgroup$
    – Mindlack
    Dec 15 '18 at 23:03










  • $begingroup$
    wich is the difference between "full Fubini Theorem" and "Fubini Theorem"?
    $endgroup$
    – Marcos Martínez Wagner
    Dec 15 '18 at 23:11










  • $begingroup$
    To me Fubini-Tonelli refers to the Fubini theorem when everything is non-negative (which is easier). By “full Fubini” I thought of the theorem when your function is not non-negative. So nothing really important.
    $endgroup$
    – Mindlack
    Dec 15 '18 at 23:13










  • $begingroup$
    Thank you!! I think I got it
    $endgroup$
    – Marcos Martínez Wagner
    Dec 15 '18 at 23:19










  • $begingroup$
    Nol, in fact the integral defining $T_f$ need not exist. (Yes if $f$ is integrable on the product space.)
    $endgroup$
    – David C. Ullrich
    Dec 15 '18 at 23:22


















  • $begingroup$
    $T_f$ is a pointwise limit of measurable functions (truncate the integral at $n$, then you can use the full Fubini theorem)
    $endgroup$
    – Mindlack
    Dec 15 '18 at 23:03










  • $begingroup$
    wich is the difference between "full Fubini Theorem" and "Fubini Theorem"?
    $endgroup$
    – Marcos Martínez Wagner
    Dec 15 '18 at 23:11










  • $begingroup$
    To me Fubini-Tonelli refers to the Fubini theorem when everything is non-negative (which is easier). By “full Fubini” I thought of the theorem when your function is not non-negative. So nothing really important.
    $endgroup$
    – Mindlack
    Dec 15 '18 at 23:13










  • $begingroup$
    Thank you!! I think I got it
    $endgroup$
    – Marcos Martínez Wagner
    Dec 15 '18 at 23:19










  • $begingroup$
    Nol, in fact the integral defining $T_f$ need not exist. (Yes if $f$ is integrable on the product space.)
    $endgroup$
    – David C. Ullrich
    Dec 15 '18 at 23:22
















$begingroup$
$T_f$ is a pointwise limit of measurable functions (truncate the integral at $n$, then you can use the full Fubini theorem)
$endgroup$
– Mindlack
Dec 15 '18 at 23:03




$begingroup$
$T_f$ is a pointwise limit of measurable functions (truncate the integral at $n$, then you can use the full Fubini theorem)
$endgroup$
– Mindlack
Dec 15 '18 at 23:03












$begingroup$
wich is the difference between "full Fubini Theorem" and "Fubini Theorem"?
$endgroup$
– Marcos Martínez Wagner
Dec 15 '18 at 23:11




$begingroup$
wich is the difference between "full Fubini Theorem" and "Fubini Theorem"?
$endgroup$
– Marcos Martínez Wagner
Dec 15 '18 at 23:11












$begingroup$
To me Fubini-Tonelli refers to the Fubini theorem when everything is non-negative (which is easier). By “full Fubini” I thought of the theorem when your function is not non-negative. So nothing really important.
$endgroup$
– Mindlack
Dec 15 '18 at 23:13




$begingroup$
To me Fubini-Tonelli refers to the Fubini theorem when everything is non-negative (which is easier). By “full Fubini” I thought of the theorem when your function is not non-negative. So nothing really important.
$endgroup$
– Mindlack
Dec 15 '18 at 23:13












$begingroup$
Thank you!! I think I got it
$endgroup$
– Marcos Martínez Wagner
Dec 15 '18 at 23:19




$begingroup$
Thank you!! I think I got it
$endgroup$
– Marcos Martínez Wagner
Dec 15 '18 at 23:19












$begingroup$
Nol, in fact the integral defining $T_f$ need not exist. (Yes if $f$ is integrable on the product space.)
$endgroup$
– David C. Ullrich
Dec 15 '18 at 23:22




$begingroup$
Nol, in fact the integral defining $T_f$ need not exist. (Yes if $f$ is integrable on the product space.)
$endgroup$
– David C. Ullrich
Dec 15 '18 at 23:22










0






active

oldest

votes











Your Answer





StackExchange.ifUsing("editor", function () {
return StackExchange.using("mathjaxEditing", function () {
StackExchange.MarkdownEditor.creationCallbacks.add(function (editor, postfix) {
StackExchange.mathjaxEditing.prepareWmdForMathJax(editor, postfix, [["$", "$"], ["\\(","\\)"]]);
});
});
}, "mathjax-editing");

StackExchange.ready(function() {
var channelOptions = {
tags: "".split(" "),
id: "69"
};
initTagRenderer("".split(" "), "".split(" "), channelOptions);

StackExchange.using("externalEditor", function() {
// Have to fire editor after snippets, if snippets enabled
if (StackExchange.settings.snippets.snippetsEnabled) {
StackExchange.using("snippets", function() {
createEditor();
});
}
else {
createEditor();
}
});

function createEditor() {
StackExchange.prepareEditor({
heartbeatType: 'answer',
autoActivateHeartbeat: false,
convertImagesToLinks: true,
noModals: true,
showLowRepImageUploadWarning: true,
reputationToPostImages: 10,
bindNavPrevention: true,
postfix: "",
imageUploader: {
brandingHtml: "Powered by u003ca class="icon-imgur-white" href="https://imgur.com/"u003eu003c/au003e",
contentPolicyHtml: "User contributions licensed under u003ca href="https://creativecommons.org/licenses/by-sa/3.0/"u003ecc by-sa 3.0 with attribution requiredu003c/au003e u003ca href="https://stackoverflow.com/legal/content-policy"u003e(content policy)u003c/au003e",
allowUrls: true
},
noCode: true, onDemand: true,
discardSelector: ".discard-answer"
,immediatelyShowMarkdownHelp:true
});


}
});














draft saved

draft discarded


















StackExchange.ready(
function () {
StackExchange.openid.initPostLogin('.new-post-login', 'https%3a%2f%2fmath.stackexchange.com%2fquestions%2f3042057%2flet-f0-infty-times-0-infty-rightarrow-bbb-r-be-a-lebesgue-measurable%23new-answer', 'question_page');
}
);

Post as a guest















Required, but never shown

























0






active

oldest

votes








0






active

oldest

votes









active

oldest

votes






active

oldest

votes
















draft saved

draft discarded




















































Thanks for contributing an answer to Mathematics Stack Exchange!


  • Please be sure to answer the question. Provide details and share your research!

But avoid



  • Asking for help, clarification, or responding to other answers.

  • Making statements based on opinion; back them up with references or personal experience.


Use MathJax to format equations. MathJax reference.


To learn more, see our tips on writing great answers.




draft saved


draft discarded














StackExchange.ready(
function () {
StackExchange.openid.initPostLogin('.new-post-login', 'https%3a%2f%2fmath.stackexchange.com%2fquestions%2f3042057%2flet-f0-infty-times-0-infty-rightarrow-bbb-r-be-a-lebesgue-measurable%23new-answer', 'question_page');
}
);

Post as a guest















Required, but never shown





















































Required, but never shown














Required, but never shown












Required, but never shown







Required, but never shown

































Required, but never shown














Required, but never shown












Required, but never shown







Required, but never shown







Popular posts from this blog

Plaza Victoria

In PowerPoint, is there a keyboard shortcut for bulleted / numbered list?

How to put 3 figures in Latex with 2 figures side by side and 1 below these side by side images but in...